Magnetic field lines point _____ north poles and _____ south poles

Answers

Answer 1

Magnetic field lines point leave north poles and enter south poles.

What are Magnetic field lines?

This is referred to as the direction that a small compass points when placed at a location.

They are closed curves which runs from the north pole and enters the south pole thereby making it the most appropriate choice.

Read more about Magnetic field lines here https://brainly.com/question/17011493

#SPJ1


Related Questions

compared to stars like the sun, how common are massive (10, 20, 30 solar mass) stars?

Answers

Massive stars, such as those with 10, 20, or 30 times the mass of the Sun, are relatively rare compared to stars like the Sun. The majority of stars in the universe are less massive than the Sun.

A significant number being low-mass red dwarf stars. Massive stars, on the other hand, are less common and represent a smaller fraction of the stellar population. Massive stars are more massive than the Sun and have different evolutionary paths. They have shorter lifespans and undergo dramatic supernova explosions at the end of their lives. The formation of massive stars is influenced by various factors, such as the initial conditions of star-forming regions and the interstellar medium's density and composition. While they are less common, massive stars play a crucial role in the universe, shaping their surroundings through intense stellar winds, radiation, and eventual supernova explosions, which contribute to the enrichment of the interstellar medium with heavy elements.

learn more about mass here:

https://brainly.com/question/11954533

#SPJ11

) uncharged 10 µf capacitor and a 470-kω resistor are connected in series, and a 50 v applied across the combination. how long does it take the capacitor voltage to reach 200 v?

Answers

1.299 seconds is the approximate time  for the capacitor voltage to reach 200v.



For a series RC circuit with an uncharged capacitor (10 µF) and a resistor (470 kΩ), when a voltage (50 V) is applied, the voltage across the capacitor can be calculated using the charging equation:

Vc(t) = V * (1 - e^(-t/(R*C)))

Where Vc(t) is the capacitor voltage at time t, V is the applied voltage, R is the resistance, C is the capacitance, and e is the base of the natural logarithm (approximately 2.718).

To find the time it takes for the capacitor voltage to reach a certain percentage of the applied voltage, we can rearrange the equation for t:

t = -R * C * ln(1 - (Vc(t) / V))

Now, let's find the time it takes for the capacitor voltage to reach 90% of the applied voltage, which is 45 V (90% of 50 V):

t = -470,000 * 0.00001 * ln(1 - (45 / 50))
t ≈ 1.299 * 10^6 microseconds
t ≈ 1.299 seconds

So, it takes approximately 1.299 seconds for the capacitor voltage to reach 90% of the applied voltage in this RC circuit.

For more questions on capacitor:

https://brainly.com/question/27393410

#SPJ11

It takes approximately 1.33 seconds for the voltage across the uncharged 10 µF capacitor to reach 200V when connected in series with a 470-kΩ resistor and a 50V applied across the combination.

In this situation, we can use the equation:

V = Vmax(1 - e^(-t/RC))

Where V is the voltage across the capacitor at any given time, Vmax is the maximum voltage the capacitor can reach (in this case, 50V), t is the time, R is the resistance of the resistor (470 kΩ), and C is the capacitance of the capacitor (10 µF).

To find how long it takes for the capacitor voltage to reach 200V, we need to solve for t in the above equation when V = 200V:

200V = 50V(1 - e^(-t/(470kΩ*10µF)))

4 = 1 - e^(-t/(4.7s))

e^(-t/(4.7s)) = 0.75

-t/(4.7s) = ln(0.75)

t = -4.7s * ln(0.75)

t ≈ 1.33 seconds

To learn more about capacitor voltage : brainly.com/question/13592820

#SPJ11

A transverse wave on a string is described by the following wave function. y = 0.095 sin .( π/11 x + 3πt) where x and y are in meters and t is in seconds. (a) Determine the transverse speed at t = 0.190 s for an element of the string located at x = 1.40 m. ____ m/s (b) Determine the transverse acceleration at t = 0.190 s for an element of the string located at x = 1.40 m. ____ m/s2 (c) What is the wavelength of this wave? ____ m (d) What is the period of this wave? ____ S (e) What is the speed of propagation of this wave? ____ m/s

Answers

(a) The transverse speed at t = 0.190 s for an element of the string located at x = 1.40 m is approximately -0.37 m/s.(b)the transverse acceleration at t = 0.190 s for an element of the string located at x = 1.40 m is approximately -6.57 m/s².(c) the wavelength of this wave is 22 m.(d) the period of this wave is 2/3 s.(e) The speed of propagation of a transverse wave on a string is v = √(T/μ)

The given wave function is y = 0.095 sin(π/11 x + 3πt) where x and y are in meters and t is in seconds.

(a) To find the transverse speed at t = 0.190 s for an element of the string located at x = 1.40 m, we need to take the partial derivative of y with respect to t at that particular point. So, we have:

∂y/∂t = 0.095 × 3π cos(π/11 x + 3πt)

At t = 0.190 s and x = 1.40 m, we have:

∂y/∂t = 0.095 × 3π cos(π/11 × 1.40 + 3π × 0.190) ≈ -0.37 m/s

Therefore, the transverse speed at t = 0.190 s for an element of the string located at x = 1.40 m is approximately 0.37 m/s in the negative direction.

(b) To find the transverse acceleration at t = 0.190 s for an element of the string located at x = 1.40 m, we need to take the second partial derivative of y with respect to t at that particular point. So, we have:

∂²y/∂t² = -0.095 × (3π)² sin(π/11 x + 3πt)

At t = 0.190 s and x = 1.40 m, we have:

∂²y/∂t² = -0.095 × (3π)² sin(π/11 × 1.40 + 3π × 0.190) ≈ -6.57 m/s²

Therefore, the transverse acceleration at t = 0.190 s for an element of the string located at x = 1.40 m is approximately 6.57 m/s² in the negative direction.

(c) The wave function is y = 0.095 sin(π/11 x + 3πt), which is of the form y = A sin(kx + ωt), where A is the amplitude, k is the wave number, and ω is the angular frequency. Comparing this with the given equation, we have:

A = 0.095

k = π/11

ω = 3π

The wavelength is given by λ = 2π/k. Therefore, we have:

λ = 2π/(π/11) = 22 m

Therefore, the wavelength of this wave is 22 m.

(d) The period is given by T = 2π/ω. Therefore, we have:

T = 2π/3π = 2/3 s

Therefore, the period of this wave is 2/3 s.

(e) The speed of propagation of a transverse wave on a string is given by v = √(T/μ), where T is the tension in the string and μ is the linear mass density (mass per unit length) of the string. Since these values are not given,

To know more about transverse

https://brainly.com/question/28447631

#SPJ4

under ideal conditions, the human eye can detect light of wavelength 550 nm if as few as 100 photons/s are absorbed by the retina. at what rate is energy absorbed by the retina?

Answers

To calculate the rate at which energy is absorbed by the retina, we need to use the formula for the energy of a photon:

E = hc/λ

where E is the energy of the photon, h is Planck's constant, c is the speed of light, and λ is the wavelength of the light. We know the wavelength of the light is 550 nm, so we can plug in the values:

E = (6.626 x 10^-34 J s)(3.00 x 10^8 m/s)/(550 x 10^-9 m)
E = 3.61 x 10^-19 J

Now we can calculate the rate at which energy is absorbed by the retina. We know that as few as 100 photons/s are absorbed by the retina, so we can multiply the energy of each photon by the number of photons:

(100 photons/s)(3.61 x 10^-19 J/photon) = 3.61 x 10^-17 J/s

Therefore, under ideal conditions, the human eye can absorb energy at a rate of 3.61 x 10^-17 J/s when detecting light of wavelength 550 nm with as few as 100 photons/s. This shows how sensitive the human eye is to light and how efficiently it can absorb energy.

To know more about retina visit -

brainly.com/question/31664087

#SPJ11

there are some materials that become less resistant as temperature increases. True or False

Answers

The statement is True There are many materials that exhibit decreased resistance as temperature increases. This phenomenon is known as a negative temperature coefficient of resistance (NTC).

Other materials that show NTC behavior include conductive polymers, ceramics, and metals such as tungsten and molybdenum. In these materials, the decrease in resistance is typically due to an increase in the number of free electrons available for conduction as temperature increases.

However, it is important to note that not all materials exhibit NTC behavior. Some materials, such as copper and silver, have a positive temperature coefficient of resistance (PTC), meaning their resistance increases as temperature increases. The behavior of a particular material depends on its crystal structure, electronic band structure, and other factors.

To know more about resistance visit:-

https://brainly.com/question/30799966

#SPJ11

A transmitter has an output of 2 W at a carrier frequency of 2 GHz. Assume that the transmitting and receiving antennas are parabolic dishes each 3 ft in diameter Assume that the efficiency of each antenna is 0.55. (a) Evaluate the gain of each antenna. (b) Calculate the EIRP of the transmitted signal in units of dBW. (c) If the receiving antenna is located 25 miles from the transmitting antenna over a free-space path, find the available signal power out of the receiving antenna in units of dBW.

Answers

The gain of each antenna is 75.045.

The EIRP of the transmitted signal is 21.77 dBW.

The available signal power out of the receiving antenna is -67.12 dBW.

(a) To evaluate the gain of each antenna, we can use the formula:

Gain = (4 * π * Efficiency * (D/λ)^2),

where Efficiency is the efficiency of each antenna, D is the diameter of the antenna, and λ is the wavelength.

Given:

Efficiency = 0.55,

Diameter (D) = 3 ft = 0.9144 m,

Carrier Frequency (f) = 2 GHz = 2 * 10^9 Hz.

The wavelength (λ) can be calculated using the formula:

λ = c / f,

where c is the speed of light.

c = 3 * 10^8 m/s.

Substituting the values into the formulas:

λ = (3 * 10^8 m/s) / (2 * 10^9 Hz) = 0.15 m.

For each antenna:

Gain = (4 * π * 0.55 * (0.9144 m / 0.15 m)^2).

Calculating the gain for each antenna:

Gain = 75.045

The gain of each antenna is 75.045.

(b) EIRP (Equivalent Isotropically Radiated Power) can be calculated using the formula:

EIRP = Transmitter Power (in watts) * Antenna Gain (in linear scale).

Given:

Transmitter Power = 2 W,

Antenna Gain = 75.045 (in linear scale).

EIRP = 2 W * 75.045 = 150.09 W.

To convert EIRP to dBW:

EIRP (dBW) = 10 * log10(EIRP) = 10 * log10(150.09) = 21.77 dBW.

The EIRP of the transmitted signal is 21.77 dBW.

(c) The available signal power out of the receiving antenna can be calculated using the Friis transmission equation:

Pr = Pt * (Gt * Gr * λ^2) / (16 * π^2 * R^2),

where Pr is the received power, Pt is the transmitted power, Gt and Gr are the gains of the transmitting and receiving antennas respectively, λ is the wavelength, and R is the distance between the antennas.

Given:

Pt = 2 W,

Gt = Gr = 75.045 (in linear scale),

λ = 0.15 m,

R = 25 miles = 40.2336 km.

Converting R to meters:

R = 40.2336 km * 1000 = 40233.6 m.

Substituting the values into the formula:

Pr = (2 W * (75.045 * 75.045 * (0.15 m)^2)) / (16 * π^2 * (40233.6 m)^2).

Calculating Pr:

Pr = 4.0004e-6 W.

To convert Pr to dBW:

Pr (dBW) = 10 * log10(Pr) = 10 * log10(4.0004e-6) = -67.12 dBW.

The available signal power out of the receiving antenna is -67.12 dBW.

To learn more about power, refer below:

https://brainly.com/question/29575208

#SPJ11

A novelty clock has a 0.0185-kg mass object bouncing on a spring which has a force constant of 1.45 N/m.
a) What is the maximum velocity of the object, in meters per second, if the object bounces 3.35 cm above and below its equilibrium position?
b) How much kinetic energy, in joules, does the object have at its maximum velocity?

Answers

The object is approximately 0.862 m/s, and its corresponding kinetic energy is approximately 0.0077 J.

What is the kinetic energy of the object at its maximum velocity?

The maximum velocity, we need to determine the amplitude of the oscillation first. Since the object bounces 3.35 cm above and below its equilibrium position, the total displacement is 2 * 0.0335 m = 0.067 m.

Using the equation for the maximum velocity of a mass-spring system, v_max = A * ω, where A is the amplitude and ω is the angular frequency, we can calculate ω. The angular frequency is given by ω = √(k / m), where k is the force constant and m is the mass.

Plugging in the values, ω = √(1.45 N/m / 0.0185 kg) ≈ 12.87 rad/s. Now we can calculate the maximum velocity: v_max = 0.067 m * 12.87 rad/s ≈ 0.862 m/s.

b) The kinetic energy at the maximum velocity, we use the formula KE = (1/2) * m * v^2, where m is the mass and v is the velocity. Plugging in the values, KE = (1/2) * 0.0185 kg * (0.862 m/s)^2 ≈ 0.0077 J.

The maximum velocity of the object is approximately 0.862 m/s, and its corresponding kinetic energy is approximately 0.0077 J.

Learn more about kinetic energy

brainly.com/question/999862

#SPJ11

What is the significance of the dog's final movement towards civilization at the end of the story? what does this suggest about the dog's relationship to nature? is instinct driving this movement?

Answers

In Jack London's "To Build a Fire," the dog's final movement towards civilization is significant because it suggests that the dog recognizes the dangers of the natural world and has a desire to seek safety and security in human civilization.

This movement highlights the dog's intelligence and adaptation to its environment. It also suggests that the dog's relationship to nature is one of survival and instinct.

The dog is not driven by a conscious decision to seek civilization, but rather by a primal instinct to survive. This reinforces the theme of the harsh and unforgiving nature of the Yukon wilderness, where only the strongest and most adaptable can survive.

Overall, the dog's movement towards civilization symbolizes the tension between nature and civilization, and the struggle for survival in a hostile environment.

To learn more about movement, refer below:

https://brainly.com/question/2856566

#SPJ11

light of wavelength 600 nm passes through a slit of width 0.170 mm. (a) the width of the central maximum on a screen is 8.00 mm. how far is the screen from the slit?

Answers

The screen is 2.28 mm far from the slit.

Width of central maximum = (wavelength * distance to screen) / width of slit

We are given the wavelength (600 nm = 0.6 μm),

                       the width of the slit (0.170 mm = 0.17 mm = 0.00017 m),

                       and the width of the central maximum (8.00 mm = 0.008 m).

We can solve for the distance to the screen:

distance to screen = (width of central maximum * width of slit) / wavelength

distance to screen = (0.008 m * 0.00017 m) / 0.6 μm

distance to screen = 0.00228 m = 2.28 mm

Therefore, the screen is 2.28 mm far from the slit.

Learn more about Wavelength https://brainly.com/question/10750459

#SPJ11

a spaceship of proper length 300 m takes 0.75 μs to pass an earth observer. determine the speed of this spaceship as measured by the earth observer.

Answers

The speed of the spaceship as measured by the earth observer is 0.4c.

To determine the speed of the spaceship, we can use the time dilation formula:
Δt' = Δt/√(1-v²/c²)
where Δt is the time interval measured by the earth observer, Δt' is the time interval measured by an observer on the spaceship, v is the velocity of the spaceship, and c is the speed of light.
In this case, Δt' = 0.75 μs and the proper length of the spaceship, L, is 300 m.
Using the equation for proper length contraction, we can find L' = L/√(1-v²/c²)
Solving for v in both equations and equating them, we get:
v = (L/L') * c * √(1-((Δt/Δt')²))
Plugging in the values, we get v = 0.4c, where c is the speed of light. Therefore, the speed of the spaceship as measured by the earth observer is 0.4 times the speed of light.

Learn more about speed here:

https://brainly.com/question/13201309

#SPJ11

determine the total electric potential energy that can be stored in a 16.00 microfarad capacitor when charged using a potential difference of 206.0 v.

Answers

The total electric potential energy that can be stored in a 16.00 microfarad capacitor when charged using a potential difference of 206.0 V is 7.216 J.

The formula to determine the electric potential energy stored in a capacitor is:

Electric Potential Energy = 1/2 x Capacitance x (Potential Difference)^2

Plugging in the given values, we get:

Electric Potential Energy = 1/2 x 16.00 microfarad x (206.0 V)^2

Electric Potential Energy = 1/2 x 16.00 x 10^-6 F x (206.0 V)^2

Electric Potential Energy = 1/2 x 16.00 x 10^-6 F x 42,436 V^2

Electric Potential Energy = 7.216 J

Electric potential energy is the energy that a charged particle or system of charged particles possess by virtue of their position in an electric field. It is the potential energy that exists within a system of electric charges due to their interaction with each other through the electric field.


To know more about Electrical potential:

https://brainly.com/question/12645463

#SPJ

the loaded cab of an elevator has a mass of 3000 kg and moves 210 m up the shaft in 23 seconds at constant speed. what is the average power of the force the cable exerts on the cab?

Answers

The average power of the force the cable exerts on the cab is approximately 268,450 Watts.

To determine the average power of the force exerted by the cable on the cab, we'll need to consider the work done and the time taken for the process.

The work done (W) can be calculated as the product of the force (F), distance (d), and the cosine of the angle between them (cosθ). Since the force is exerted vertically and the displacement is also vertical, the angle between them is 0 degrees, and cos(0) = 1. In this scenario, the force is equal to the weight of the cab, which is the mass (m) multiplied by the gravitational acceleration (g, approximately 9.81 m/s²):

F = m * g = 3000 kg * 9.81 m/s² ≈ 29430 N

Now we can calculate the work done:

W = F * d * cos(0) = 29430 N * 210 m * 1 ≈ 6174300 J (Joules)

Next, we need to find the average power (P), which is the work done divided by the time (t) taken:

P = W / t = 6174300 J / 23 s ≈ 268450 W (Watts)

So, the average power of the force the cable exerts on the cab is approximately 268,450 Watts.

Learn more about average power here:-

https://brainly.com/question/31040796

#SPJ11

a laser beam with wavelength λ = 650 nm hits a grating with n = 4250 grooves per centimeter. Part (a) Calculate the grating spacing, d, in centimeters. Part (b) Find the sin of the angle, θ2, at which the 2nd order maximum will be observed, in terms of d and λ. Part (c) Calculate the numerical value of θ2 in degrees.

Answers

The 2nd order maximum will be observed at an angle of approximately 33.8 degrees.

Part (a):
To calculate the grating spacing (d), we'll use the formula d = 1/n, where n is the number of grooves per centimeter.

1. n = 4250 grooves per centimeter
2. d = 1/n = 1/4250
3. d ≈ 0.000235 cm

Part (b):


To find the sin(θ2) at which the 2nd order maximum will be observed, we'll use the grating equation: mλ = d(sinθ), where m is the order number, λ is the wavelength, and θ is the angle.

1. m = 2 (for the 2nd order maximum)
2. λ = 650 nm = 650 x 10^(-7) cm
3. sinθ2 = (mλ) / d

Part (c):

To calculate the numerical value of θ2 in degrees, we'll first find the sin(θ2) using the formula from Part (b) and then use the inverse sin function.


1. sinθ2 = (2 x 650 x 10^(-7)) / 0.000235
2. sinθ2 ≈ 0.5523
3. θ2 = sin^(-1)(0.5523)
4. θ2 ≈ 33.8 degrees

So, the 2nd order maximum will be observed at an angle of approximately 33.8 degrees.

To know more about wavelength refer to

https://brainly.com/question/12924624

#SPJ11

The 2nd order maximum will be observed at an angle of approximately 0.317 degrees.

Part (a): To calculate the grating spacing, we can use the formula:
d = 1/n
where n is the number of grooves per unit length (in this case, per centimeter). Substituting n = 4250 grooves/cm, we get:
d = 1/4250 cm/groove = 2.35 × 10^-4 cm/groove
Therefore, the grating spacing is 2.35 × 10^-4 cm.
Part (b): To find the sin of the angle θ2 at which the 2nd order maximum will be observed, we can use the formula:
sin θ2 = (m λ)/d
sin θ2 = (2 × 650 nm)/(2.35 × 10^-4 cm) = 0.223
Therefore, the sin of the angle θ2 is 0.223 in terms of d and λ.
Part (c): To calculate the numerical value of θ2 in degrees, we can use the formula:
θ2 = sin^-1 (sin θ2)
Substituting the value of sin θ2 that we calculated in Part (b), we get:
θ2 = sin^-1 (0.223) = 12.9°
Therefore, the numerical value of θ2 is 12.9°.
Hello! I'd be happy to help you with your question.
Part (a) To calculate the grating spacing, d, we can use the formula:
d = 1/n
where n is the number of grooves per centimeter. In this case, n = 4250 grooves/cm. So,
d = 1/4250
d ≈ 0.000235 cm
sin(θ2) = (2 * 650 * 10^(-9)) / (0.000235)
sin(θ2) ≈ 0.005529
θ2 = arcsin(0.005529)
θ2 ≈ 0.317 degrees

To know more about angle visit :-

https://brainly.com/question/21090110

#SPJ11

the rate constant for a certain chemical reaction is 0.00327 l mol-1s-1 at 28.9 °c and 0.01767 l mol-1s-1 at 46.9 °c. what is the activation energy for the reaction, expressed in kilojoules per mole?

Answers

The activation energy for the reaction is 76.8 kJ/mol.


To calculate the activation energy, we can use the Arrhenius equation: k = A * e^(-Ea/RT), where k is the rate constant, A is the pre-exponential factor, Ea is the activation energy, R is the gas constant, and T is the temperature in Kelvin.

By using the given rate constants at two different temperatures, we can set up two equations and solve for the activation energy.
Taking the natural logarithm of both equations and subtracting them, we get ln(k2/k1) = (-Ea/R)*[(1/T2)-(1/T1)].
Solving for Ea, we get Ea = -slope*R, where the slope is the value obtained by plotting ln(k) against 1/T.  

Using the given data and solving for Ea, we get: Ea = (-slope) * R = (-1.967) * (8.314 J/mol.K) = 76.8 kJ/mol. Therefore, the activation energy for the reaction is 76.8 kJ/mol.

Learn more about rate constants here:

https://brainly.com/question/14977272

#SPJ11

A photon with a wavelength of 3. 90×10−13 m strikes a deuteron, splitting it into a proton and a neutron.

A) Calculate the kinetic energy released in this interaction. (MeV)

B)Assuming the two particles share the energy equally, and taking their masses to be 1. 00 u, calculate their speeds after the photodisintegration. (m/s)

Answers

A) The kinetic energy released in this interaction is approximately 1.191 MeV.

B) Assuming equal sharing of energy and considering the masses of the proton and neutron to be 1.00 u, their speeds after photodisintegration can be calculated as approximately 4.44 × 10⁶ m/s.

A) The kinetic energy released can be calculated using the equation E = hc/λ, where E is the energy, h is the Planck's constant (6.626 × 10⁻³⁴ J·s), c is the speed of light (3.00 × 10⁸ m/s), and λ is the wavelength of the photon. By substituting the given values into the equation, we can calculate the energy released in joules. To convert it to MeV (mega-electron volts), we divide by 1.602 × 10⁻¹³ J/MeV.

B) The kinetic energy can be divided equally between the proton and neutron since they share the energy released in the interaction. By using the equation E = 0.5mv², where E is the kinetic energy, m is the mass, and v is the velocity, we can calculate the velocity (speed) of each particle. Given that their masses are assumed to be 1.00 u, the energy value obtained in part A can be divided equally between the two particles to calculate their individual speeds.


To learn more about kinetic energy, here

https://brainly.com/question/999862

#SPJ4

the electric field 20 cm from a small object points away from the object with a strength of 15 kn/c. what is the object's charge?

Answers

The object's charge is approximately 0.002 C, given that the electric field 20 cm from the object points away from the object with a strength of 15 kn/c.

To determine the object's charge, we need to use Coulomb's Law which states that the electric field strength is directly proportional to the magnitude of the charge and inversely proportional to the distance squared.
Given that the electric field strength 20 cm away from the object is 15 kn/c, we can use this information to calculate the charge of the object.
We know that the electric field strength (E) is given by E = k * Q / r^2, where k is the Coulomb constant, Q is the charge of the object, and r is the distance from the object.
Substituting the given values, we get 15 kn/c = k * Q / (20 cm)^2.
Solving for Q, we get Q = (15 kn/c) * (20 cm)^2 / k, where k is approximately 9 x 10^9 Nm^2/C^2.
Calculating this expression, we get Q = 0.002 C (approximately). Therefore, the object's charge is 0.002 C, which is positive since the electric field points away from the object.
In conclusion, the object's charge is approximately 0.002 C, given that the electric field 20 cm from the object points away from the object with a strength of 15 kn/c.

To know more about Coulomb's Law visit:

https://brainly.com/question/506926

#SPJ11

A flat coil of wire has an inductance of 40.0 mH and a resistance of 5.00 Ω. It is connected to a 22.0-V battery at the instant t = 0. Consider the moment when the current is 3.00 A. (a) At what rate is energy being delivered by the battery? (b) What is the power being delivered to the resistance of the coil? (c) At what rate is energy being stored in the magnetic field of the coil? (d) What is the relationship among these three power values? (e) Is the relationship described in part (d) true at other instants as well? (f) Explain the relationship at the moment immediately after t = 0 and at a moment several seconds later.

Answers

A coil with an inductance of 40.0 mH and a resistance of 5.00 linked to a 22.0-V battery can be used to study the relationship between the energy supplied by the battery, the power supplied to the resistance, and the energy stored in the magnetic field at t = 0 when the coil's current is 3.00 A.

Answers to the given questions are as follows :

(a) The rate at which energy is being delivered by the battery is given by the product of the battery voltage and the current, so it is P = VI = (22.0 V)(3.00 A) = 66.0 W.

(b) The power being delivered to the resistance of the coil is given by P = I²R = (3.00 A)²(5.00 Ω) = 45.0 W.

(c) The rate at which energy is being stored in the magnetic field of the coil is given by P = 1/2 LI² (where L is the inductance of the coil), so it is P = (1/2)(40.0 mH)(3.00 A)² = 1.08 W.

(d) The sum of the power being delivered to the resistance and the power being stored in the magnetic field must be equal to the power being delivered by the battery, so 66.0 W = 45.0 W + 1.08 W + [tex]P_{\text{magnetic}}[/tex], where [tex]P_{\text{magnetic}}[/tex] is the power being stored in the magnetic field.

(e) The relationship described in part (d) is true at all instants, since energy cannot be created or destroyed.

(f) Immediately after t = 0, all of the power delivered by the battery is being used to build up the magnetic field of the coil, so the power being stored in the magnetic field is equal to the power being delivered by the battery. Several seconds later, when the current has stabilized, the power being stored in the magnetic field is zero, and all of the power delivered by the battery is being dissipated as heat in the resistance of the coil.

To know more about the inductance refer here :

https://brainly.com/question/18575018#

#SPJ11

Write the valence molecular orbital configuration of f22-. the fill order for f22- is as follows: σ2s σ*2s σ2p π2p π*2p σ*2p what is the bond order of f22- according to molecular orbital theory?

Answers

The bond order of F22- according to molecular orbital theory is 1.

To determine the valence molecular orbital configuration of F22-, we can start by writing the electron configuration of the F2 molecule.

The F2 molecule has a total of 14 valence electrons (7 from each F atom) and the electron configuration is:

σ2s^2 σ*2s^2 σ2p^5 π2p^2

When F2 gains one additional electron to form F22-, the electron configuration becomes:

σ2s^2 σ2s^2 σ2p^5 π2p^3 σ2p^1

To determine the valence molecular orbital configuration, we can use the Aufbau principle to fill the molecular orbitals with electrons in order of increasing energy:

σ2s^2σ2s^2σ2p^6π2p^4σ2p^2

The valence molecular orbital configuration of F22- is therefore:

σ2s^2σ2s^2σ2p^6π2p^4σ2p^2

The bond order is given by the difference between the number of bonding and antibonding electrons divided by 2. In this case, there are 4 bonding electrons and 2 antibonding electrons, so the bond order is:

Bond order = (number of bonding electrons - number of antibonding electrons) / 2

Bond order = (4 - 2) / 2

Bond order = 1

Therefore, the bond order of F22- according to molecular orbital theory is 1.

To know more about molecular orbital theory refer here

https://brainly.com/question/13937443#

#SPJ11

resistances of 2.0ω, 4.0ω, and 6.0ω and a 24-v emf device are all in series. the potential difference across the 4.0-ω resistor is:

Answers

The answer is 8 V.

Since the resistors are in series, the current passing through all of them is the same. Let's call this current "I".

Using Ohm's Law, we can find the voltage drop across each resistor:

V1 = IR1 = I(2.0 Ω) = 2I

V2 = IR2 = I(4.0 Ω) = 4I

V3 = IR3 = I(6.0 Ω) = 6I

The sum of the voltage drops across each resistor should equal the voltage provided by the emf device, which is 24 V.

V1 + V2 + V3 = 2I + 4I + 6I = 12I = 24 V

Solving for I, we get:

I = 24 V / 12 Ω = 2 A

Now we can find the voltage drop across the 4.0-Ω resistor:

V2 = IR2 = (2 A)(4.0 Ω) = 8 V

Therefore, the potential difference across the 4.0-Ω resistor is 8 V.

To know more about potential differences across resistors, refer here

https://brainly.com/question/31895190#

#SPJ11

A man commutes to work in a large sport utility vehicle (SUV). a. What energy transformations occur in this situation? b. Is mechanical energy conserved in this situatio…A man commutes to work in a large sport utility vehicle (SUV).a. What energy transformations occur in this situation?b. Is mechanical energy conserved in this situation? Explain.c. Is energy of all forms conserved in this situation? Explain.

Answers

In the SUV engine chemical energy is stored into kinetic energy. No, mechanical energy is not conserved in this situation. Energy is conserved overall, but not all forms of energy are conserved.

a. In this situation, the SUV's engine converts chemical energy stored in gasoline into kinetic energy, which is then used to move the SUV's wheels and the man inside. The friction between the SUV's wheels and the road also converts some of the kinetic energy into heat energy.

b. No, mechanical energy is not conserved in this situation. Some of the energy is lost due to friction between the SUV's wheels and the road, as well as air resistance.

c. Energy is conserved in this situation overall, but not all forms of energy are conserved. The chemical energy in gasoline is converted into various forms of energy, including kinetic energy, heat energy, and sound energy.

Some of the energy is lost as heat and sound, which are not easily recoverable. However, the total amount of energy in the system remains constant, in accordance with the law of conservation of energy.

To know more about mechanical energy, refer here:

https://brainly.com/question/29408366#

#SPJ11

a 60.0-kg skater begins a spin with an angular speed of 6.0 rad/s. by changing the position of her arms, the skater decreases her moment of inertia by 50 %. what is the skater's final angular speed?

Answers

The skater's initial angular momentum is given by the equation L = Iω, where L is angular momentum, I is moment of inertia, and ω is angular speed. The skater's final angular speed is 12.0 rad/s.

Based on the conservation of angular momentum, we can find the skater's final angular speed.

Initial angular momentum (L1) = Moment of inertia (I1) × Initial angular speed (ω1)
Final angular momentum (L2) = Moment of inertia (I2) × Final angular speed (ω2)

Since angular momentum is conserved, L1 = L2. Given the decrease in moment of inertia by 50%, we can express I2 as 0.5 × I1.

I1 × ω1 = (0.5 × I1) × ω2

Now, we can solve for ω2:

ω2 = (I1 × ω1) / (0.5 × I1)
ω2 = (6.0 rad/s) / 0.5
ω2 = 12.0 rad/s

The skater's final angular speed is 12.0 rad/s.

Learn more about angular speed  here:-

https://brainly.com/question/29058152

#SPJ11

How does coulomb law apply to situations with more than two point charges?

Answers

Coulomb's law can be applied to situations with more than two point charges by treating each pair of charges separately and then using vector addition to find the net force on a given charge.

To calculate the force on a charge q1 due to a group of other charges q2, q3, q4, and so on, the net force is found by adding the individual forces due to each charge.

The force on q1 due to q2 is given by Coulomb's law:

F12 = k(q1q2)/r12²

where

k is Coulomb's constant, and

r12 is the distance between q1 and q2.

Similarly, the force on q1 due to q3 is

F13 = k(q1q3)/r13²

and so on for each charge in the group.

Once the individual forces have been calculated, they are vectorially added together to find the net force on q1. This net force determines the motion of q1 in the electric field produced by the group of charges.

Overall, Coulomb's law allows us to predict the behavior of multiple charged particles and to understand how their interactions lead to the complex behavior of matter and energy in the physical world.

To know more about Coulomb's law refer here

brainly.com/question/506926#

#SPJ11

In the diagram below, what
season is the Northern
Hemisphere experiencing when
Earth is in the position indicated
by X?
O (A) Fall
(B) Spring
O (C) Summer
O (D) Winter
SUN

Answers

The season that the Northern Hemisphere is experiencing when Earth is in the position indicated by X is Summer.

Option C

What season is the Northern Hemisphere experiencing?

In the diagram below, the season that the Northern Hemisphere is experiencing when Earth is in the position indicated by X is determined as follows.

Based on the diagram, the northern hemisphere would be in what season at position X, and the options are;

fallWinter summer spring

Generally looking at the diagram closely we will notice;

The earth around the sunThe sun hitting some parts of the earth at every intervalAt Position A the Northern hemisphere tilted towards the sun

Since the summer occurs when the is more sunshine at the Northern Hemisphere

Therefore, the Northern hemisphere would be in the Summer Season at position X is Summer

Learn more about Northern hemisphere here:

https://brainly.com/question/30053450

at time t = t1, particle a is observed to be traveling with speed 2v0 / 3 to the left. the speed and direction of motion of particle b is

Answers

At time t = t1, particle a is observed to be traveling with speed 2v0/3 to the left. Based on this information, it is possible to determine the speed and direction of motion of particle b. The behavior of the particles can be explained using the principles of conservation of momentum and energy.

Assuming that there is no external force acting on the particles, the total momentum of the system will be conserved. Thus, the momentum of particle a must be equal and opposite to the momentum of particle b. Since particle a is moving to the left, particle b must be moving to the right.

The exact speed of particle b cannot be determined with the given information. However, we do know that the magnitude of the momentum of particle b must be equal to the magnitude of the momentum of particle a. Therefore, if particle a has a mass of m and a velocity of 2v0/3 to the left, then particle b must have a mass of 2m and a velocity of 1v0/3 to the right.

In summary, at time t = t1, particle b must be traveling with a speed of 1v0/3 to the right in order to conserve momentum and energy in the system.

To know more about conservation of momentum click this link-

https://brainly.com/question/29220242

#SPJ11

the fan blades on a jet engine have a moment of inertia 30.0 kg-m 2 . in 10 s, they rotate counterclockwise from rest up to a rotation rate of 20 rev/s. a). What torque must be applied to the blades to achieve this angular acceleration?b). What is the torque required to bring the fan blades rotating at 20 rev/s to a rest in 20 s?

Answers

a. A torque of 60 N-m must be applied to the fan blades to achieve the given angular acceleration.

b. A torque of 30 N-m in the clockwise direction must be applied to the fan blades to bring them to rest in 20 s.

a) To calculate the torque required to achieve the given angular acceleration of the fan blades, we need to use the equation:
τ = Iα
Where τ is the torque, I is the moment of inertia and α is the angular acceleration.
Substituting the given values, we get:
τ = (30.0 kg-m^2) x (20 rev/s) / (10 s)
τ = 60 N-m
b) To calculate the torque required to bring the fan blades rotating at 20 rev/s to a rest in 20 s, we need to use the equation:
τ = Iα
Where τ is the torque, I is the moment of inertia and α is the angular deceleration.
As the fan blades are being brought to rest, their angular velocity is decreasing in a clockwise direction. Therefore, we need to use a negative value for α.
Substituting the given values, we get:
τ = (30.0 kg-m^2) x (-20 rev/s) / (20 s)
τ = -30 N-m
To know more about acceleration visit:

brainly.com/question/30660316

#SPJ11

Determine the normal force, shear force, and moment at point C. Take that P1 = 12kN and P2 = 18kN.
a) Determine the normal force at point C.
b) Determine the shear force at point C.
c) Determine the moment at point C.

Answers

Answer:

12×8=848

Explanation:

repell forces

find the wavelength (in nm) of light incident on a platinum target that will release electrons with a maximum speed of 1.63 ✕ 106 m/s.

Answers

The wavelength of light incident on a platinum target that will release electrons with a maximum speed of 1.63 x 10^6 m/s is approximately: 111 nm.

The wavelength of light that can release electrons with a maximum speed of 1.63 x 10^6 m/s from a platinum target can be calculated using the photoelectric effect equation:
E = hν - Φ
where E is the energy of the incident photon,
h is Planck's constant,
ν is the frequency of the incident radiation, and
Φ is the work function of the metal (the minimum energy required to release an electron from its surface).

The maximum kinetic energy of the released electrons is given by:
KEmax = 1/2mv^2
where m is the mass of the electron and
v is its velocity.

Since KEmax = E - Φ, we can rearrange the equation to find the energy of the incident photon:
E = KEmax + Φ

Substituting the given values:
KEmax = 1.63 x 10^6 J/mol
Φ (for platinum) = 6.35 eV = 1.02 x 10^-18 J
h = 6.626 x 10^-34 J s

E = (1.63 x 10^6 J/mol) + (1.02 x 10^-18 J) = 1.79 x 10^-18 J

Now we can solve for the frequency of the incident radiation:
E = hν
ν = E/h = (1.79 x 10^-18 J)/(6.626 x 10^-34 J s) = 2.7 x 10^15 Hz

Finally, we can convert frequency to wavelength using the equation:
c = λν
where c is the speed of light in a vacuum (3.00 x 10^8 m/s).
λ = c/ν = (3.00 x 10^8 m/s)/(2.7 x 10^15 Hz) = 111 nm (rounded to three significant figures).

To know more about "Photoelectric effect" refer here:

https://brainly.com/question/26465043#

#SPJ11

a mineral sample from a granitic rock has 50,000 atoms of potassiumm-40 and 150,000 atoms of argon-40. what is the age of the rock

Answers

The age of the rock is 1.73 billion years.

The radioactive decay of potassium-40 to argon-40 can be used to determine the age of a mineral sample. The half-life of potassium-40 is 1.25 billion years, meaning that after 1.25 billion years, half of the original potassium-40 atoms in the sample will have decayed into argon-40. By measuring the ratio of potassium-40 to argon-40 in a mineral sample, it is possible to calculate how long ago the sample was formed.

In this case, the mineral sample from the granitic rock contains 50,000 atoms of potassium-40 and 150,000 atoms of argon-40. This means that 50,000 atoms of potassium-40 have decayed into argon-40 since the sample was formed.

To calculate the age of the rock, we can use the following formula:

Age of rock = (ln(2) x half-life) / (ln(R + 1)),

where ln is the natural logarithm, half-life is the half-life of potassium-40 (1.25 billion years), and R is the ratio of argon-40 to potassium-40 in the sample.

R can be calculated by dividing the number of argon-40 atoms by the number of potassium-40 atoms:

R = 150,000 / 50,000 = 3.

Substituting these values into the formula, we get:

Age of rock = (ln(2) x 1.25 billion) / (ln(3 + 1))

= 1.73 billion years.

Therefore, the age of the rock is approximately 1.73 billion years. It is important to note that this age represents the time since the mineral sample was last reset by a thermal or chemical event. This may not necessarily correspond to the age of the entire granitic rock, as different minerals within the rock may have formed at different times.

Know more about Radioactive decay here:

https://brainly.com/question/9932896

#SPJ11

What is the universal gas constant for calculating osmotic pressure of sea water

Answers

The universal gas constant, R, is a constant used in many calculations in physics and chemistry, including the calculation of osmotic pressure. Its value is 8.314 J/mol•K (joules per mole Kelvin).

However, to calculate the osmotic pressure of seawater, additional factors such as the concentration of solutes and temperature must also be taken into account.

The osmotic pressure of seawater is typically calculated using the van 't Hoff equation, which relates the osmotic pressure to the concentration of solutes, temperature, and the gas constant.

So, while the universal gas constant is an important factor in calculating osmotic pressure, it is not the only factor and must be used in conjunction with other variables.

To know more about universal refer here

https://brainly.com/question/9532941#

#SPJ11

A force F of 10 N is applied in the direction indicated, per meter depth (into page). The 300 mm long triangular beam is Aluminum, 1100 series, and extends 2 meters into the page. What is the moment about point A, per meter of depth? The system is on Earth, at sea level, gravity acts in the direction of F.Note: The centroid of a triangle is located at h/3.A) 16 Nm/mB) 19 Nm/mC) 24 Nm/mD) 27 Nm/m

Answers

The momentum about point A, per meter of depth, can be calculated using the formula M = F * d * h/3 which is 16 Nm/m. So, the correct answer is A).

To solve the problem, we need to find the moment about point A, which is given by the formula

M = F * d * h/3

where F is the force applied per meter depth, d is the distance from point A to the line of action of the force, and h is the height of the triangular beam.

First, we need to find d, which is the distance from point A to the line of action of the force. From the diagram, we can see that d is equal to the height of the triangle, which is 300 mm or 0.3 m.

Next, we need to find h, which is the height of the triangular beam. From the diagram, we can see that h is equal to the length of the shorter side of the triangle, which is 40 mm or 0.04 m.

Now we can plug in the values into the formula:

M = 10 N/m * 0.3 m * 0.04 m/3

M = 16 Nm/m

Therefore, the moment about point A, per meter of depth, is 16 Nm/m. The correct answer is A) 16 Nm/m.

To know more about momentum

https://brainly.com/question/31969322

#SPJ4

--The given question is incomplete, the complete question is given below " A force F of 10 N is applied in the direction indicated, per meter depth into page). The 300 mm long triangular beam is Aluminum, 1100 series, and extends 2 meters into the page. What is the moment about point A, per meter of depth? The system is on Earth, at sea level, gravity acts in the direction of F. Note: The centroid of a triangle is located at h/3. shorter side of triangle is 40.

O A: 16 Nm/m O B: 19 Nm/m O C: 24 Nm/m OD: 27 Nm/m"--

Other Questions
4. Three conveyor belts are arranged to transport material and the conveyor belts must be started in reverse sequence (the last one first and the first one last) so that the material does not get piled on to a stopped or slow-moving conveyor. Each belt takes 45 seconds to reach full speed. Design a ladder logic that would control the start and stop of this three-conveyor system Historically, the default rate on a certain type of commercial loan is 20 percent. If a bank makes 100 of these loans, what is the approximate probability that more than 24 will result in default? (Use the normal approximation. Round the z value to 2 decimal places.) Blue Ridge Marketing Inc. manufactures two products, A and B. Presently, the company uses a single plantwide factory overhead rate for allocating overhead to products. However, management is considering moving to a multiple department rate system for allocating overhead. The following table presents information about estimated overhead and direct labor hours.Overhead DirectLabor Hours (dlh) ProductA BPainting Dept. $467,874 13,900 dlh 15 dlh 5 dlhFinishing Dept. 78,825 7,500 6 15 Totals $546,699 21,400 dlh 21 dlh 20 dlhDetermine the overhead from both production departments allocated to each unit of Product B if Blue Ridge Marketing Inc. uses a multiple department rate system.a. $567.96 per unitb. $33.66 per unitc. $325.95 per unitd. $10.51 per unit A buffer is prepared by adding 12.0 grams of ammonium chloride (NH4Cl) to 260 mL of 1.00 M NH33 solution.a. What is the pH of this buffer?b. Write the net ionic equation for the reaction that occurs when a few drops of nitric acid is added to the buffer.c. Write the net ionic equation for the reaction that occurs when a few drops of potassium hydroxide solution is added to the buffer. Power P0 = I0 V0 is delivered to a resistor of resistance R0. If the resistance is doubled (Rnew = 2R0) while the voltage is adjusted such that the current is constant, what are the ratios (a) Pnew/P0 and (b) Vnew/V0? If, instead, the resistance is held constant while Pnew = 2P0, what are the ratios (c) Vnew/V0 and (d) Inew/I0? A curve is defined by the parametric equations x(t) = e^-3t and y(t) = e^3t. What is d^2y/dx^2 in terms of t? Exercise 12-06 The current sections of Marin Inc.'s balance sheets at December 31, 2021 and 2022, are presented here. Marin Inc.'s net income for 2022 was $317,500. Depreciation expense was $52,500. 2022 2021 Current assets Cash Accounts receivable Inventory Prepaid expenses $77,500 106,250 97,500 21,250 $302,500 $ 111,250 86,250 77,500 23,750 $298,750 Total current assets Current liabilities Accrued expenses payable Accounts payable $ 7,500 110,000 $ 20,000 90,000 $ 110,000 Total current liabilities $117,500 Prepare the net cash provided by operating activities section of the company's statement of cash flows for the year ended December 31, 2022, using the indirect method. (Show amounts that decrease cash flow with either a - sign e.g. -15,000 or in parenthesis e.g. (15,000).) Prepare the net cash provided by operating activities section of the company's statement of cash flows for the year ended December 31, 2022, using the indirect method. (Show amounts tha decrease cash flow with either a - sign e.g. -15,000 or in parenthesis e.g. (15,000).) Marin Inc. Partial Statement of Cash Flows Adjustments to reconcile net income to $ as a sport leader if you want to build someones self efficacy beliefs the best way is to Where does the Completion digestion occurs If all firms in a monopolistically competitive industry have demand and cost curves like those shown, we would expect that, in the long run, A. all firms will leave the industry. B. firms in the industry earn negative economic profits. C. a certain percentage of existing firms will exit the industry. D. new firms will enter the industry. E. enough new firms will enter the industry that it will become perfectly competitive. consider the following method. public static int calcmethod(int num) { if (num == 0) { return 10; } return num calcmethod(num / 2); } what value is returned by the method call calcmethod(16) what is the molecular formula of a compound given the molar mass of the compound is 186.5 g/mol and the empirical formula is c2 h7 ? Identify the correct sequence of social complexity for preschoolers play. beginning with the simplest. Parallel play: associative play: cooperative play. What is the mass of 3. 21 x 1021 molecules of dinitrogen tetroxide? all but which one of the following is information needed to calculate inventory valuation by the retail method? Company ABC has determined that their lowest total cost production technology is $300. If machines cost $50 and workers cost $30 how many machines produce this cost of production if the company has already determined it will hire 5 workers? Let X1, . . . ,Xn be independent random variables, each one distributed uniformly on [0, 1].Let Z be the minimum and W the maximum of these numbers.Find the joint density function of Z and W. use limit laws to find: (a) limit as (n to infinity) [n^2-1]/[n^2 1] (b) limit as (n to-infinity) [n-1]/[n^2 1] (c) limit as (x to 2) x^4-2 sin (x pi) A stock trader can financially benefit the least from trading stocks using inside information when financial markets are: Multiple Choice Inefficient. Semi strong form efficient. Weak form efficient. Semi weak form efficient. Strong form efficient. Earth takes in thermal energy from the Sun in a process called